Anyone?

bclnxx(x+a)dx\large{\displaystyle \int^{c}_{b} \frac{\ln x}{\sqrt{x}(x+a)}dx}

#Calculus

Note by Tanishq Varshney
5 years, 9 months ago

No vote yet
1 vote

  Easy Math Editor

This discussion board is a place to discuss our Daily Challenges and the math and science related to those challenges. Explanations are more than just a solution — they should explain the steps and thinking strategies that you used to obtain the solution. Comments should further the discussion of math and science.

When posting on Brilliant:

  • Use the emojis to react to an explanation, whether you're congratulating a job well done , or just really confused .
  • Ask specific questions about the challenge or the steps in somebody's explanation. Well-posed questions can add a lot to the discussion, but posting "I don't understand!" doesn't help anyone.
  • Try to contribute something new to the discussion, whether it is an extension, generalization or other idea related to the challenge.
  • Stay on topic — we're all here to learn more about math and science, not to hear about your favorite get-rich-quick scheme or current world events.

MarkdownAppears as
*italics* or _italics_ italics
**bold** or __bold__ bold

- bulleted
- list

  • bulleted
  • list

1. numbered
2. list

  1. numbered
  2. list
Note: you must add a full line of space before and after lists for them to show up correctly
paragraph 1

paragraph 2

paragraph 1

paragraph 2

[example link](https://brilliant.org)example link
> This is a quote
This is a quote
    # I indented these lines
    # 4 spaces, and now they show
    # up as a code block.

    print "hello world"
# I indented these lines
# 4 spaces, and now they show
# up as a code block.

print "hello world"
MathAppears as
Remember to wrap math in \( ... \) or \[ ... \] to ensure proper formatting.
2 \times 3 2×3 2 \times 3
2^{34} 234 2^{34}
a_{i-1} ai1 a_{i-1}
\frac{2}{3} 23 \frac{2}{3}
\sqrt{2} 2 \sqrt{2}
\sum_{i=1}^3 i=13 \sum_{i=1}^3
\sin \theta sinθ \sin \theta
\boxed{123} 123 \boxed{123}

Comments

Well I am getting this -

1a[zlog(z)z+a+zlog(z)2aΦ(za,1,12)zaΦ(za,1,12)]bc\displaystyle \dfrac{1}{a} \left[\frac{\sqrt{z} log(z)}{z + a} + \frac{\sqrt{z} log(z)}{2a} \Phi\left(-\frac{z}{a}, 1, \frac{1}{2}\right) - \frac{\sqrt{z}}{a} \Phi\left(-\frac{z}{a}, 1, \frac{1}{2}\right)\right]_b^c

Kartik Sharma - 5 years, 9 months ago

Log in to reply

ok frankly i didnt understand what u have typed here. anyway i was trying to find

116lnxx(x+4)dx\large{\displaystyle \int^{16}_{1} \frac{\ln x}{\sqrt{x}(x+4)}dx}

Tanishq Varshney - 5 years, 9 months ago

Log in to reply

Substitute x=y2 x = y^2 . Then put y=2ty=\dfrac{2}{t} to again get the same integral and evaluate it. I'll post the complete solution if you want.

Sudeep Salgia - 5 years, 9 months ago

Log in to reply

@Sudeep Salgia yup plz

Tanishq Varshney - 5 years, 9 months ago

Log in to reply

@Tanishq Varshney Let I=116lnxx(x+4)dx \displaystyle I = \int_1^{16} \frac{ \ln x }{\sqrt{x}(x+4) } dx . Substitute x=y2 x = y^2 to get I=144lny(y2+4)dy \displaystyle I = \int_1^{4} \frac{ 4 \ln y }{(y^2+4) } dy . Now put y=4t y = \dfrac{4}{t} to get I4=14ln4lnt(4+t2)dt \displaystyle \frac{I}{4} = \int_1^4 \frac{ \ln 4 - \ln t}{(4 + t^2)} dt . Rearranging, we get, I=2ln4141t2+4dt \displaystyle I = 2 \ln 4 \int_1^4 \frac{1}{t^2 + 4} dt .
This is a simple integration. So the answer becomes, I=ln4(arctan(2)arctan(1/2)) I = \ln 4 ( \arctan (2) - \arctan(1/2))

Hope this helps. Please check and let me know if there are any calculation mistakes. (I wasn't writing any thing down :P )

Sudeep Salgia - 5 years, 9 months ago

Log in to reply

@Sudeep Salgia Clever substitution!

Aditya Kumar - 5 years, 9 months ago

@Sudeep Salgia Can you please explain the method where you divide I/4 . And when you re-arrange ln (t) disappears ?

Thanks.

Syed Baqir - 5 years, 9 months ago

But I need to check once again.

Kartik Sharma - 5 years, 9 months ago
×

Problem Loading...

Note Loading...

Set Loading...